Difference between revisions of "2021 AMC 10B Problems/Problem 1"
Line 2: | Line 2: | ||
<math>\textbf{(A)} ~9 \qquad\textbf{(B)} ~10 \qquad\textbf{(C)} ~18 \qquad\textbf{(D)} ~19 \qquad\textbf{(E)} ~20</math> | <math>\textbf{(A)} ~9 \qquad\textbf{(B)} ~10 \qquad\textbf{(C)} ~18 \qquad\textbf{(D)} ~19 \qquad\textbf{(E)} ~20</math> | ||
+ | |||
+ | ==Solution== | ||
+ | |||
+ | Since <math>3\pi</math> is about <math>9.42</math>, we multiply 9 by 2 and add 1 to get <math> \boxed{\textbf{(D)}\ ~9} </math> |
Revision as of 16:35, 11 February 2021
How many integer values of satisfy ?
Solution
Since is about , we multiply 9 by 2 and add 1 to get